LSAT and Law School Admissions Forum

Get expert LSAT preparation and law school admissions advice from PowerScore Test Preparation.

User avatar
 KelseyWoods
PowerScore Staff
  • PowerScore Staff
  • Posts: 1079
  • Joined: Jun 26, 2013
|
#72639
Complete Question Explanation

Strengthen-Principle. The correct answer choice is (B).

We are being asked to find a principle that would strengthen the argument in the editorial. So the first thing we need to figure out is: what is the argument in the editorial? The conclusion is the first statement, that the government should not use lottery revenue to fund any of its health services. Why does the author argue that we shouldn't use the lottery revenue for health services? Because, according to the author, health services are essential to the community and lottery revenue is not reliable. With Strengthen-Principle questions, we're basically trying to find an abstract rule that the specific argument in the stimulus adheres to. So, to prephrase, we are looking for a principle in the answer choices that strengthens the argument by providing an abstract rule that basically says that if it's an unreliable revenue source, it shouldn't be used to support an essential government service.

Answer choice (A): The argument in the stimulus is not about how much governments should spend on essential services vs. nonessential services. Instead, the argument is about what funding sources we should use for essential services. So answer choice (A) is irrelevant to this argument.

Answer choice (B): This is the correct answer choice. This answer choice fits our prephrase nicely and does a good job of providing an abstract rule that our argument mirrors. If essential government services must be funded from reliable sources of revenue, this strengthens the idea that the government should not use the lottery to fund health services because it is an unreliable revenue source.

Answer choice (C): This answer choice is close to what we're looking for but not quite there. We're trying to support the conclusion that the lottery should not fund any part of the government's health services. This principle just says that government services shouldn't be entirely dependent on lottery revenue. We need a principle to more specifically address the conclusion above that lottery revenue shouldn't be used to fund any part of the government's health services, not just the entirety of it.

Answer choice (D): This answer choice does nothing to strengthen the conclusion because it has nothing to do with whether lottery revenue should not be used to fund any part of the health services.

Answer choice (E): This answer choice also does nothing to strengthen the conclusion. Setting aside lottery revenue in the case of budget shortfalls does not strengthen the conclusion that lottery revenue should not be used to fund the health services.
 mjb514
  • Posts: 35
  • Joined: Nov 01, 2017
|
#41970
Can you please explain this problem, and why C is wrong, thanks.
 Luke Haqq
PowerScore Staff
  • PowerScore Staff
  • Posts: 747
  • Joined: Apr 26, 2012
|
#42020
HI mjb514,

The editorial in this question is making an argument about a government's health services. Namely, the editorial first argues (1) that these health services are important and (2) that they shouldn't be funded by lottery revenues.

To the question, why shouldn't they be funded by lottery revenues?, the editorial provides an answer: " lottery revenue could decline at some time in the future, leaving the government scrambling to make up a budget shortfall."

In other words, the editorial is against using lottery revenues as funds or health services specifically because they are unpredictable, which could leave the government "scrambling" to try to find funds to pay for health services. Clearly the author, then, finds this scrambling and unpredictability to be bad (e.g., people might not get its health services, if the government discovers it can't pay for them). And this is what answer (B) gets across: "Essential government services must be funded from reliable sources of revenue." The author recommends against lottery revenues because they're unpredictable/unreliable, so (B) reflects the implicit principle at work in the author's argument.

Answer (C) states that "No government service should be entirely dependent on lottery revenue for its funding." This couldn't be a principle the author is relying on because the author's conclusion is "The government should not fund any part of its health services with lottery revenue." Answer (C) is a principle that governments shouldn't be entirely dependent on lottery revenue--i.e., this language means (C) is fine with the government being somewhat dependent on lottery revenues. Again, the conclusion is that governments should not in any way at all rely on lottery revenues for their health services, so (C) can't be an implicit principle underlying that argument.
 T.B.Justin
  • Posts: 194
  • Joined: Jun 01, 2018
|
#61786
I also think the word "essential" is important when analyzing this stimulus and incorrect answer choice (C), since the stimulus is referring to health services as essential (important).
 Brook Miscoski
PowerScore Staff
  • PowerScore Staff
  • Posts: 418
  • Joined: Sep 13, 2018
|
#63333
T.B. Justin,

The word "essential" is irrelevant to why (B) is superior to (C) and every other choice. (B) would work just as well if "essential" was omitted. (C) is wrong because the stimulus states that no part of funding should come from the lottery. (C) would allow the lottery to be part of the funding, as long as there were also other sources.
 haganskl
  • Posts: 43
  • Joined: May 30, 2019
|
#79862
Hello.

Would C be correct if it read,
“No government service should be dependent on lottery revenue for its funding.”?

I eliminated this answer choice because the stim referred to “no” govt service. After reading through this thread, I believe I didn’t eliminate it for the right reason. I understand why C is wrong now. Just curious.

TIA!
 Adam Tyson
PowerScore Staff
  • PowerScore Staff
  • Posts: 5153
  • Joined: Apr 14, 2011
|
#81082
While that modified answer would strengthen the conclusion of the argument, haganskl, it would not, in my opinion, do what the question stem is really asking us for. We need a rule that not only helps the conclusion, but that the argument followed structurally. This author did not build an argument based solely on the lottery being a bad source - the argument was built around the idea of the lottery being a source that could at some point dry up.

When we talk about an argument conforming to a principle, we mean that it follows that rule in a structural sense. Think of it as a conditional relationship - we need a rule that has a sufficient condition and a necessary condition, and the argument either says "the necessary condition must occur here because the sufficient condition occurred," or else "the sufficient condition cannot occur in this case because the necessary condition did not." The sufficient condition in the argument (the premise) is that the lottery revenues might decline in the future, so we need an answer that incorporates that concept. Your suggested change to answer C, while it helps support the conclusion, still doesn't deal with that conditional relationship, which the stem is asking us to do.

Thankfully in this case we don't have to make that kind of close call, because answer C is even worse as written, while answer B is exactly what we need.

Get the most out of your LSAT Prep Plus subscription.

Analyze and track your performance with our Testing and Analytics Package.